NSG 6020 Week 1-9 Quiz With Complete Solutions
1. BUN
Answer: requires adequate protein ingestion
2. serum creatinine levels
Answer: • requires adequate muscle mass
• unreliable in older adults
3. topical medication absorption
Answer: much higher in infants and young children due to thin
4. stratum
Answer: leads to potential systemic toxicity
5. infants younger than 6 months
Answer: • decreased plasma proteins for drug binding—increased plasma levels of drug
• avoid NSAIDs and monitor for drug toxicity
6. paediatric OTC considerations
Answer: • no ASA
• no alternation between APAP and ibuprofen in outpatient setting
7. state board of nursing
Answer: determine legal APN scope of practice and prescriptive authority
8. FDA, NPI, DEA
Answer: federal level agencies affecting prescribing
9. factors contributing to scheduled drugs
Answer: currently accepted medical use in treatment in
10. US relative abuse potential
Answer: likelihood of causing dependence when abused
11. schedule 1 drugs -no accepted medical use in the US high potential for abuse
Answer: heroin, LSD, ecstasy
12. schedule 2 drugs - high potential for abuse which may lead to severe psychological or
physical dependence
Answer: Dilaudid, Demerol, oxycodone, fentanyl, morphine, codeine, hydrocodone,
amphetamines
13. schedule 3 drugs - potential for abuse less than substances in schedule 1 or 2 and abuse may
lead to moderate or low physical dependence or high psychological dependence
Answer: products not containing 90 mg of codeine per dosage unit, suboxone, Tylenol w/
codeine, testosterone
14. schedule 4 drugs - low potential for abuse relative to substances in schedule 3
Answer: Xanax, Soma, Klonipin, Valium, Ativan, Versed, Restoril, Halcion
15. schedule 5 drugs - low potential for abuse related to substances in schedule 4 and consist
primary of preparations containing limited quantities of certain narcotics
Answer: • cough preparations not containing more than 200mg of codeine per 100 ml or per 100
grams
• Robitussin AC or Phenergan w/ Codeine
16. medication error
Answer: any mistake occurring in the medication use process, regardless of whether an injury
occurred or whether the potential for injury was present
17. adverse drug event
Answer: any harmful/undesirable event resulting from med use
18. categories of adverse drug events
Answer: • preventable
• non-preventable--adverse reactions
• potential--near miss errors or close calls
19. drug classes consistently associated with ADEs
Answer: • anticoagulants
• diabetes meds
• antibiotic
• sedatives
• narcotics
• antipsychotics
• chemo agents
20. MedWatch- FDA Safety Info and Adverse Event Reporting program
Answer: voluntarily report a serious adverse event, product quality problem, product use error,
or therapeutic inequivalence/failure that you suspect is associated with use of FDA regulated
drug, biologic, medical device, dietary supplement or cosmetic
21. Strategies to prevent ADE
Answer: • review med list and allergies with each encounter
• be vigilant with high-risk drugs
• discontinue unnecessary drugs
• make sure patients are clear on new medications if intended to add or replace old med
• avoid treating side effects with another drug
• address nonadherence
• EMR/escribe
22. hypertriglyceridemia
Answer: Fish oil indication
23. depression
Answer: St. John's wort indication
24. dementia
Answer: Ginkgo biloba indication
25. heart failure
Answer: CoQ10 indication
26. antibiotic associated diarrhea
Answer: probiotic indication
27. most common medication malpractice errors
Answer: • prescribing wrong med
• prescribing wrong dose
• Failing to properly discontinue a med
• Prescribing an incompatible, contraindicated, interactive med
28. adherence
Answer: extent to which a person's behavior--taking medication, following a diet, or making
healthy lifestyle changes--corresponds with agreed-upon recommendations from a healthcare
provider
29. Medication adherence
Answer: Patient's conformance with the provider's recommendation with respect to medication:
• Timing
• Dosage
• Frequency
• For prescribed length of time
30. high risk for medication non-adherence
Answer: • asymptomatic diseases (HTN)
• cognitive impairment
• psychiatric illness
• complex regimens
31. common non-adherent behaviours
Answer: • took less often
• smaller doses
• stopped
• sooner delayed
• fill failed to fill
32. barriers to medication adherence
Answer: • perceived/actual adverse reactions
• asymptomatic conditions
• knowledge deficit/health literacy
• health beliefs
• cognitive impairment and psychiatric illness
• communication difficulties
• cost
• access
• chronic conditions (esp >3)
• complexity of drug regimen
• multiple providers
• recent hospital discharge
33. SIMPLE
Answer: • Simplify the regimen
• Impart knowledge
• Modify patient beliefs and behavior
• Provide communication and trust
• Leave the bias
• Evaluate adherence
34. pharmacokinetics
Answer: • drug absorption, distribution, binding affinity, metabolism, and excretion going in and
coming out of body
• what body does to drug
35. pharmacodynamics
Answer: • interactions within body tissue
• what drug does to body
36. pharmacogenetics
Answer: • variability in drug response/toxicity due to genetic factors
• how our DNA affects the way drugs work
• seeks to guide the design of optimal medication regimens for individual patients
37. genetic polymorphisms in targets of drug therapy
Answer: can greatly influence efficacy and toxicity of medications
38. pharmacogenomics
Answer: • development and use of medications
• applying genetic info to deciding which drug to use
39. 3 phases of drug action
Answer: 1. pharmaceutical--how drug comes out of delivery form--breakdown of tablet/capsule
2. pharmacokinetic
3. pharmacodynamic
40. basic pharmacokinetic processes
Answer: 1. absorption--with or without food
2. distribution--protein binding/hypoalbuminemia
3. metabolism--primarily liver
4. elimination--urine and feces
41. best absorbed drug form
Answer: non-ionized, lipid-soluble form
42. water-soluble drugs
Answer: cannot pass through lipid membranes
43. lipid-soluble drugs
Answer: more easily pass through cell membranes CNS drugs
44. high ionization
Answer: • more difficult time crossing lipid membranes
• acid drug and base media or base drug and acid media
45. ileum
Answer: most drugs absorbed here d/t microvilli--increased surface area
46. albumin
Answer: most common site of plasma protein binding
47. Warfarin
Answer: 97% protein bound drug--only 3% free drug
48. urine elimination
Answer: requires drug to be water-soluble--if not must be bio transformed
49. CYP450 enzyme locations
Answer: liver, intestines, lungs, other organs
50. CYP450 inducer
Answer: • speeds up enzyme activity
• increased metabolism=decreased drug level
Examples:
• phenobarbs
• Rifampin
• cigarette smoke
• anticonvulsants
51. CYP450 inhibitor
Answer: • decreases enzyme activity
• decreased metabolism=increased drug level
Examples:
• cimetidine
• omeprazole
• macrolides
• antifungals
• St. John's Wort
52. grapefruit juice - common CYP450 inhibitor
Answer: after 1 glass, statin level doubled
53. urine excretion
Answer: increased ionization=increased excretion
54. weak base drug+acidic urine
Answer: increased excretion of amphetamines
55. creatinine clearance
Answer: • Cockcroft-Gault used to estimate
• can overestimate in frail elders
• package inserts base renal dosing on
56. half-life
Answer: time for plasma concentration of drug to be reduced by 50%
57. hepatic first-pass effect
Answer: • amount of drug liver metabolizes on first pass before drug gets to blood stream in oral
administration
• can be bypassed by IV, transdermal, rectal delivery
58. prodrug
Answer: administered drug is metabolized to another active drug with intended purpose
decreased liver fxn can prevent conversion to active/desired drug
59. metabolite
Answer: end product of metabolism--can be active or inactive
Prilosec-->Nexium
60. zero-order elimination
Answer: drug is metabolized at constant amount per unit of time, regardless of concentration
61. set number of drug receptors available
Answer: when receptor enzymes become saturated, more drug is being put in than can be
metabolized--increased blood levels of drug
62. first-order elimination
Answer: constant proportion of the drug is eliminated per unit of time 50% of drug eliminated
with each half-life
63. linear process
Answer: • 5: number of half-lives needed to reach steady state
• number of half-lives needed to completely eliminate drug
64. steady state
Answer: each administered dose produces consistent increase and decrease in plasma drug
concentration
65. therapeutic range
Answer: The range within which a drug's concentration is effective without causing toxicity
66. loading dose
Answer: administering large amount of drug to immediately achieve therapeutic steady state
67. drug agonist
Answer: binds to receptor and produces effect
68. drug antagonist
Answer: binds to receptor and produces no effect
69. partial agonist
Answer: binds to receptor and produces suboptimal effect
70. affinity
Answer: attraction of drug to receptor
71. tolerance
Answer: loss of receptor response and drug effect
72. tachyphylaxis
Answer: rapid development of tolerance--occurs after 1st or 2nd dose
73. potency
Answer: 1. sensitivity of receptor to drug
2. give less drug for same optimal effect
74. efficacy
Answer: maximum effect
75. ED50
Answer: dose produces a half maximal response
76. LD50
Answer: dose produces a half toxic response
77. therapeutic index
Answer: LD50/ED50
78. want wide therapeutic index
Answer: drugs require lab monitoring with narrow therapeutic index
79. adrenergic drug side effects
Answer: • increased BP
• increased HR
• stroke
• mydriasis
• cardiac arrhythmias
• increased blood glucose
• uterine relaxation
80. alpha blocker side effects
Answer: HTN
81. beta blocker side effects
Answer: • increased blood glucose
• increased blood lipids bradycardia
• fatigue
• bronchoconstriction
82. Cholinergic side effects
Answer: • Salivation
• Lacrimation
• Urination
• Defecation
• GI distress
• Emesis
• sludge
83. anticholinergic side effects
Answer: • dry mouth
• blurred vision
• urinary retention
• constipation
• flushing
• tachycardia
• confusion, amnesia, hallucinations (esp in elderly)
84. new pregnancy labeling
Answer: • will contain info on: pregnancy, lactation, males/females of reproductive potential
• risk summary, clinical considerations, data
85. Beers list
Answer: • patients 65 and up
• high concern: drugs avoided due to ineffectiveness or unnecessary high risk for older persons
and a safer alternative is available
• low concern: should not be used in older persons with specific medical conditions
Examples:
long acting benzos, sedatives, long acting oral hypoglycaemics, analgesics, antiemetics, GI
antispasmodics
86. herbal supplement label
Answer: This product has not been evaluated by FDA and is not intended to diagnose, treat, cure
or prevent any disease
87. concerns with herbal supplements
Answer: • lack of FDA standards
• regarded as food sources instead of drugs
• no regulation on purity, labeling, safety, uniformity
• has grown significantly in US
88. CYP interactions
Answer: grapefruit juice and other drug inhibitors or inducers make most impact if ingested at
same time or within 4 hours of drug
89. melatonin
Answer: • used for insomnia
• exogenous consumption can suppress melatonin production from pineal gland
• uses same GABAnergic mechanism as benzos
• not recommended more than 3 nights/week
90. St. John's wort
Answer: • can interact with many other
• drugs avoid in children, pregnancy, lactation
91. Red yeast rice
Answer: • hyperlipidaemia
• contraindicated in pregnancy and lactation
92. start low and go slow
Answer: major geriatric prescribing consideration
93. increased potential for drug toxicity
Answer: lower serum albumin levels in geriatric patients
94. renal function
Answer: not easily established in older adults
95. allergic drug reactions
Answer: • anaphylaxis
• angioedema/urticaria
• serum
• sickness SJS
• TEN
• fixed drug eruptions
96. pregnancy category A
Answer: remote risk of fetal harm
RCTs in women have been done and failed to demonstrate risk of fetal harm during first
trimester, no evidence of risk in later trimesters
97. pregnancy category B
Answer: • slightly more risk than A
• animal studies show no fetal risk but no RCTs have been performed in women OR
• animal studies do show risk of fetal harm but RCTs have failed to demonstrate risk during first
trimester and no evidence of risk in later trimesters
98. pregnancy category C
Answer: • greater risk than B
• animal studies show risk of fetal harm but no RCTs done in women OR
• no studies done in women or animals
99. pregnancy category D
Answer: • proven risk of fetal harm
• studies in women show proof of fetal damage but potential benefits of use in pregnancy may be
acceptable despite risks
• statement will appear in WARNING section of drug labeling
100. pregnancy category X
Answer: • proven risk of fetal harm
• studies in women or animals show definite risk of fetal abnormality OR
• adverse reaction reports indicate evidence of fetal risk
• risks clearly outweigh any possible benefit
• statement will appear in contraindications section of drug labelling
101. beta agonist
Answer: produces fight or flight response
102. beta antagonist
Answer: blocks fight or flight response
NSG 6020 WEEK 2 QUIZ 1
1. A 29-year-old computer programmer comes to your office for evaluation of a headache. The
tightening sensation is located all over the head and is of moderate intensity. It used to last
minutes, but this time it has lasted for 5 days. He denies photophobia and nausea. He spends
several hours each day at a computer monitor/keyboard. He has tried over the-counter
medication; it has dulled the pain but not taken it away. Based on this description, what is your
most likely diagnosis?
Answer: Tension
2. Which of the following is a symptom involving the eye?
Answer: scotomas
3. A sudden, painless unilateral vision loss may be caused by which of the following?
Answer: Retinal detachment
4. Very sensitive methods for detecting hearing loss include which of the following?
Answer: Audiometric testing
5. A college student presents with a sore throat, fever, and fatigue for several days. You notice
exudates on her enlarged tonsils. You do a careful lymphatic examination and notice some
scattered small, mobile lymph nodes just behind her sternocleidomastoid muscles bilaterally.
What group of nodes is this?
Answer: Posterior cervical
6. You feel a small mass that you think is a lymph node. It is mobile in both the up-and-down and
side-to-side directions. Which of the following is most likely?
Answer: lymph node
7. A 15-year-old high school sophomore comes to the clinic for evaluation of a 3-week history of
sneezing; itchy, watery eyes; clear nasal discharge; ear pain; and nonproductive cough. Which is
the most likely pathologic process?
Answer: allergic
8. A 37-year-old nurse comes for evaluation of colicky right upper quadrant abdominal pain. The
pain is associated with nausea and vomiting and occurs 1 to 2 hours after eating greasy foods.
Which one of the following physical examination descriptions would be most consistent with the
diagnosis of cholecystitis?
Answer: Abdomen is soft and tender to palpation in the right upper quadrant with inspiration, to
the point of stopping inspiration, and there is no rebound or guarding.
9. You are interviewing an elderly woman in the ambulatory setting and trying to get more
information about her urinary symptoms. Which of the following techniques is not a component
of adaptive questioning?
Answer: Reassuring the patient that the urinary symptoms are benign and that she doesn't need
to worry about it being a sign of cancer
10. which of the following is used to screen for colour blindness in a seven year old boy
Answer: ishihara test
11. For which of the following patients would a comprehensive health history be appropriate?
Answer: A new patient with chief complaint of "I am here to establish care"
12. The following information is recorded in the health history: "The patient has had abdominal
pain for 1 week> The pain last for 30 minutes at a time: it comes and goes. The severity is 7 to 9
on a scale of 1 to 10. It is accompanied by nausea and vomiting. It is located in the midepigastric area: which category does it belong
Answer: Present illness
13. The following information is recorded in the health history: " I feel really tire" which
category does it belong
Answer: Chief complaint
14. The following information is recorded in the health history: " Patient denies chest pain,
palpitations, orthopnoea, and paroxysmal nocturnal dyspnea: which category does it belong
Answer: Review of Systems (ROS)
15. The following information is best place in which category "the patient has had three
caesarean sections."
Answer: surgeries
NSG 6020 WEEK 3 FINAL
1. In examining a 70-year-old male patient, the nurse notices that he has bilateral gynecomastia.
Which of the following describes the nurse's best course of action?
Answer: Explain that this condition may be the result of hormonal changes and recommend that
he see his physician.
2. While the nurse practitioner is taking the history of a 68-year-old patient who sustained a head
injury 3 days earlier, he tells the nurse practitioner that he is on a cruise ship and is 30 years old.
The nurse practitioner knows that this finding is indicative of:
Answer: Decreased level of consciousness
3. A woman has just been diagnosed with HPV, or genital warts. The nurse should counsel her to
receive regular examinations because this virus makes her at a higher risk for
Answer: cancer. - cervical
4. During an annual check-up of a 55 y/o patient, the nurse discusses the early detection
measures for colon cancer. The nurse should mention the need for a
Answer: colonoscopy every 10 years
5. What are the two main parts of the nervous system?
Answer: Central Nervous System (CNS) and Peripheral Nervous System (PNS)
6. While obtaining a history of a 3-month old infant from the mother, the nurse practitioner asks
about the baby's ability to suck and grasp the mother's finger. What is the nurse practitioner
assessing?
Answer: Rooting and sucking reflex. (reflexes)
7. The nurse practitioner auscultates a functional systolic murmur, grade II/IV, on a woman in
week 30 of her pregnancy. The remainder of her physical assessment is within normal limits. The
nurse practitioner would:
Answer: Know that this is a normal finding resulting from the increase in blood volume during
pregnancy.
8. The nurse practitioner knows that classic symptoms associated with preeclampsia include:
Answer: elevated blood pressure and proteinuria
9. Which of the following are changes associated with menopause
Answer: uterine and ovarian atrophy along with thinning vaginal epithelium
10. A 65-year-old patient remarks that she just can't believe that her breasts sag so much. She
states it must be from lack of exercise. What explanation should the nurse offer her?
Answer: After menopause, the glandular and fat tissue atrophies, causing breast size and
elasticity to diminish, resulting in breast that sag
11. The mother of a 10-year-old boy asks the nurse to discuss the recognition of puberty. The
nurse should reply by saying?
Answer: the first sign of puberty is enlargement of the testes
12. A patient has bilateral pitting edema of the feet. While assessing the peripheral vascular
system, the nurse's primary focus should be:
Answer: venous function of the lower extremities
13. During an examination, the nurse notes severe nystagmus in both eyes of a patient. Which of
the following conclusions is correct?
Answer: this may indicate disease of the cerebellum or brainstem
14. When performing a musculoskeletal assessment, the nurse knows the correct approach for the
examination should be:
Answer: proximal to distal
15. A 43-year-old woman is at the clinic for a routine examination. She reports that she has had a
breast lump in her right breast for years. Recently, it has begun to change in consistency and is
becoming harder. She reports that 5 years ago her physician evaluated the lump and determined
that it "was nothing to worry about." The examination validates the presence of a mass in the
right appear upper outer quadrant at 1 o'clock, approximately 5 cm form the nipple. It is firms,
mobile, nontender, with borders that are not well defined. The nurse's recommendation to her is:
Answer: Because of the change in consistency of the lump, it should be further evaluated by a
physician
16. The nurse practitioner is examining a 3-month-old infant. While holding the thumbs on the
infant's inner-mid-thighs and the fingers outside on the hips, touching the greater trochanter, the
nurse practitioner adducts the legs until the nurse practitioner's thumbs touch and then abducts
the legs until the infant's knees touch the table. The nurse practitioner does not note any
"clunking" sounds and is confident to record a:
Answer: Negative Ortolani's sign
17. A patient's mother has noticed that her son, who has been to a new babysitter, has some
blisters and scabs on his face and buttocks. On examination, the nurse notices moist, thin- roofed
vesicles with a thin erythematous base and suspects:
Answer: Impetigo
18. The nurse is testing superficial reflexes on an adult patient. When stroking up the lateral side
of the sole and across the ball of the foot, the nurse notices the plantar flexion of the toes. How
would the nurse document this finding?
Answer: Plantar reflex present
19. A woman is in the family planning clinic seeking birth control information. She states that
her breasts "change all month long" and that she is worried that this is unusual. What is the
nurse's best response?
Answer: Tell her that, because of the changing hormones during the monthly menstrual cycle,
cyclic breast changes are common.
20. A patient states during the interview that she noticed a new breast lump in the shower a few
days ago. It was on her left breast near her axilla. The RN should plan to:
Answer: palpate the unaffected breast first
21. A 16-yr-old girl is being seen at clinic for gastrointestinal complaints and weight loss. Nurse
determines that many of her complaints may be related to erratic eating patterns, eating
predominantly fast foods, + high caffeine intake. In this situation, which is most appropriate
when collecting current dietary intake information?
Answer: • Have the patient complete a food diary for 3 days=2 weekdays + 1 weekend day
Explanation:
• Food diaries require the individual to write down everything consumed for a certain time
period. Because of the erratic eating patterns of this individual, assessing dietary intake over a
few days would produce more accurate information regarding eating patterns.
• Direct observation is best used with young children or older adults.
22. To assess the head control of a 4-month-old infant, the nurse lifts the infant up in a prone
position while supporting his chest. The nurse looks for what normal response?
Answer: Raises head and arches back.
23. A patient has a positive Homans' sign. The nurse knows that a positive Homans' sign may
indicate:
Answer: deep vein thrombosis
24. The nurse is discussing breast self-examination with a postmenopausal woman. The best time
for postmenopausal women to perform breast self-examination is:
Answer: the same day every month
25. A 15-year-old boy is seen in the clinic for complaints of "dull pain and pulling" in the scrotal
area. On examination the nurse palpates a soft, irregular mass posterior to and above the testis on
the left. This mass collapses when the patient is supine and refills when he is upright. This
description is consistent with:
Answer: varicocele
26. The nurse is bathing an 80-year-old man and notices that his skin is wrinkled, thin, lax, and
dry. This finding would be related to which factor?
Answer: An increased loss of elastin and a decrease in subcutaneous fat in the elderly
27. A woman who is 22 weeks pregnant has a vaginal infection. She tells the nurse that she is
afraid that this infection will hurt the fetus. The nurse knows that which of these statements is
true?
Answer: A thick mucus plug forms that protects the fetus from infection
28. In assessment of 1-month-old, nurse notes a lack of response to noise or stimulation. mother
reports that in the last week he has been sleeping all the time + when awake all he does is cry.
nurse hears that infant's cries are very high pitched and shrill. What would be nurse's appropriate
response?
Answer: Refer the infant for further testing
29. A female patient is 8 months pregnant. She comments that she has noticed a change in
posture and is having lower back pain. The nurse tells her that during pregnancy women have a
posture shift to compensate for the enlarging fetus. This shift in posture is:
Answer: lordosis
30. A 14-year-old girl is anxious about not having reached menarche. When taking the history,
the nurse should ascertain which of the following?
Answer: The age: she began to develop breasts
31. A 9-year-old girl is in the clinic for a sports physical. After some initial shyness she finally
asks, "Am I normal? I don't seem to need a bra yet, but I have some friends who do. What if I
never get breasts?" The nurse's best response would be:
Answer: "I understand that it is hard to feel different from your friends. Breasts usually develop
between 8 and 10 years of age."
32. During an examination, the nurse notes a supernumerary nipple just under the patient's left
breast. The patient tells the nurse that she always thought it was a mole. Which statement about
this finding is correct?
Answer: It is a normal variation and not a significant finding
33. The major artery that supplies blood to arm
Answer: brachial artery
34. The nurse is testing the function of cranial nerve XI. Which of these best describes the
response the nurse should expect if the nerve is intact?
Answer: The patient moves head and shoulder against resistance with equal strength
35. In a person with an upper motor neuron lesion such as a cerebrovascular accident, which of
the following physical assessment findings would the nurse expect to see?
Answer: Hyperreflexia
36. A few days after a summer hiking trip, a 25-year-old man comes to the clinic with a rash. On
examination, the nurse notes that the rash is red, macular, with a bull's eye pattern across his
midriff and behind his knees. The nurse suspects:
Answer: Lyme disease
37. To assess the dorsalis pedis artery, the nurse would palpate
Answer: lateral to the extensor tendon of the great toe. The dorsalis pedis artery is located on the
dorsum of the foot. Palpate just lateral to and parallel with the extensor tendon of the big toe.
38. The nurse is performing a neurologic assessment on a 41-year-old woman with a history of
diabetes. When testing her ability to feel the vibrations of a tuning fork, the nurse notes the
following: unable to feel vibrations on the great toe or ankle bilaterally; is able to feel vibrations
on both patellae. Given this information, what would the nurse suspect?
Answer: Peripheral neuropathy
39. The nurse is reviewing an assessment of a patient's peripheral pulses and notices that the
documentation states that the radial pulses are "2+." The nurse recognizes that this reading
indicates what type of pulse?
Answer: normal
40. During an examination, you note that a male patient has a red, round, superficial ulcer with a
yellowish-serous discharge on his penis. Upon palpation, you note a nontender base that feels
like a small button between your thumb and fingers. At this point you suspect that this patient
has:
Answer: syphilitic chancre
41. During an interview, a patient reveals that she is pregnant. She states that she is not sure
whether she will breastfeed her baby and asks for some information about this. Which of these
statements by the nurse is accurate with regard to breastfeeding?
Answer: Breastfeeding provides the perfect food and antibodies for your baby."
42. To palpate the temporomandibular joint, the nurse's finger should be placed in the depression
of the ear.
Answer: anterior to the tragus
43. The nurse practitioner is examining only the rectal area of a woman and should place the
woman in what position?
Answer: The nurse should place the female patient in lithotomy position If examining genitalia
as well; use the left lateral decubitus position for the rectal area alone.
44. The nurse is performing a genitourinary assessment on a 50-year-old obese male laborer. On
examination the nurse notices a painless round swelling close to the pubis in the area of the
internal inguinal ring that is easily reduced when the individual is supine. These findings are
most consistent with a(n) hernia.
Answer: direct inguinal
45. The nurse is conducting a class about breast self-examination (BSE). Which of these
statements indicates proper BSE technique?
Answer: The best time to perform BSE is 4 to 7 days after the first day of the menstrual period.
46. A man found wandering in park at 2AM has been brought to emergency department for
examination because he said he fell and hit head. During examination, nurse asks him to use his
index finger to touch nurse's finger, then own nose, then nurse's finger again (moved to a
different location). patient is clumsy, unable to follow the instructions, and overshoots mark,
missing finger. nurse suspects
Answer: Acute alcohol intoxication
47. Which of the following assessment findings is most consistent with clubbing of the
fingernails?
Answer: An angle of the nail base of 180 degrees or greater with a nail base that feels spongy
48. A 45-year-old mother of two children is seen at the clinic for complaints of "losing my urine
when I sneeze." The nurse documents that she is experiencing:
Answer: stress incontinence
49. A patient calls the clinic for instructions before having a Papanicolaou (Pap) smear. The most
appropriate instructions from the nurse are:
Answer: "Avoid intercourse, inserting anything into the vagina, or douching within 24 hours of
your appointment."
50. Which of the following statements is true regarding the penis?
Answer: The corpus spongiosum expends into a cone of erectile tissue called the glans
51. During an assessment, the nurse notes that a patient's left arm is swollen from the shoulder
down to the fingers, with nonpitting edema. The right arm is normal. The patient had a
mastectomy 1 year ago. The nurse suspects which problem?
Answer: lymphedema
52. When performing the bimanual examination, the nurse notices that the cervix feels smooth
and firm, is round, and is fixed in place (does not move). When cervical palpation is performed,
the patient complains of some pain. The nurse's interpretation of these results should be which of
these?
Answer: The cervix should move when palpated; an immobile cervix may indicate malignancy.
53. You are examining a 6 month old baby. You place the baby's feet flat on the table and flex his
knees up. you note that the right knee is significantly lower than the left. Which of the following
is true of this finding?
Answer: This is a positive Allis sign and suggests hip dislocation This is a positive Allis sign and
suggests hip dislocation
54. A 75-year-old woman who has a history of diabetes and peripheral vascular disease has been
trying to remove a corn on the bottom of her foot with a pair of scissors. The nurse will
encourage her to stop trying to remove the corn with scissors because:
Answer: the woman could be at increased risk for infection and lesions because of her chronic
disease
55. You are examining Mr. O, and when you ask him to bend forward from the waist, you notice
lateral tilting; when you raise his leg straight up, he complains of a pain going down his buttock
into his leg. You suspect:
Answer: herniated nucleus pulposus
56. The assessment of an 80-year-old patient, the nurse notes that his hands show tremors when
he reaches for something and that his head is always nodding. There is no associated rigidity
with movement. Which of the following statements is most accurate?
Answer: these findings are normal
57. Which of the following statements is true regarding the arterial system?
Answer: The arterial system is a high-pressure system
58. A patient who is visiting the clinic complains of having "stomach pains for 2 weeks" and
describes his stools as being "soft and black" for about the last 10 days. He denies taking any
medications. The NP is aware that these symptoms are most indicative of:
Answer: occult blood resulting from gastrointestinal bleeding.
59. During an internal examination of a woman's genitalia, the nurse will use which technique
for proper insertion of the speculum?
Answer: Instruct the woman to bear down, turn the width of the blades horizontally, and insert
the speculum at a 45-degree angle downward toward the small of the woman's back.
60. During an internal examination of a woman's genitalia, the nurse practitioner will use which
technique for proper insertion of the speculum?
Answer: Insert the blades of the speculum on a horizontal plane, turning them to a 45-degree
angle while continuing to insert them. Ask the woman to bear down to ease insertion.
61. A 2-year-old boy has been diagnosed with "physiologic cryptorchidism." Given this
diagnosis, during assessment the nurse will most likely observe:
Answer: An absence of the testis in the scrotum, but the testis can be milked down
62. The nurse practitioner is doing an assessment on a 29-year-old woman who visits the clinic
complaining of "always dropping things and falling down." While testing rapid alternating
movements, the nurse practitioner notices that the woman is unable to pat both her knees. Her
response is very slow and she misses frequently. What might the nurse practitioner suspect?
Answer: Dysfunction of the cerebellum In rapid, alternating movements, slow, clumsy, and
sloppy response occurs with cerebellar disease.
63. When assessing a patient the nurse practitioner documents the left femoral pulse as 0/0- 4+.
Which of the following findings would the nurse practitioner expect at the dorsalis pedis pulse?
Answer: 0/0-4+ Pulsations are graded on a four-point scale: 0, absent; 1+, weak; 2+, normal; 3+,
increased; 4+, bounding. If a pulse is absent at the femoral site, one would expect the dorsalis
pedis pulse to be absent also.
64. The wife of a 65 year old man tells the nurse that she is concerned because she has noted a
change in her husband's personality and ability to understand. He also cries and becomes angry
very easily. The nurse recalls that the cerebral lobe responsible for these behaviours is which of
the following?
Answer: frontal
65. A male patient with possible fertility problems asks the nurse where sperm is produced. The
nurse knows that sperm production occurs in the:
Answer: testes
66. Which of the following statements reflects the best approach to teaching a woman about
breast self-examination (BSE)?
Answer: BSE on a monthly basis will help you feel familiar with your own breasts and their
normal variations."
67. When assessing a patient's pulse, the nurse practitioner notes that the amplitude is weaker
during inspiration and stronger during expiration. When the nurse practitioner measures the
blood pressure, the reading decreases 20 mm Hg during inspiration and increases with
expiration. This patient is experiencing:
Answer: pulsus paradoxus
68. A patient has had a "terrible itch" for several months that he has been scratching
continuously. On examination, the nurse might expect to find:
Answer: lichenification
69. During an annual physical exam, a 43-year-old patient states that she doesn't perform
monthly breast self-examination (BSE). She tells the nurse that she believes that mammograms
"do a much better job than I ever could to find a lump." The nurse should explain to her that:
Answer: BSEs may detect lumps that appear between mammograms./ mammography may not
detect all palpable lumps.
70. A patient's annual physical examination reveals a lateral curvature of the thoracic and lumbar
segments of his spine; however, this curvature disappears with forward bending. This
abnormality of the spine would be called:
Answer: Functional scoliosis
71. Assessment of a 60-yr-old patient has taken longer than anticipated. in testing pain perception
nurse decides to complete the test as quickly as possible. When nurse applies sharp point of pin
on his arm several times, he is only able to identify these as one 'very sharp prick.' most accurate
explanation?
Answer: most likely the result of the summation effect.
72. Assessing a 7-month-old infant you make a loud noise and note the following response:
Abduction and flexion of arms and legs; fanning of fingers and curling of index finger and thumb
in C-position; followed by infant bringing in arms and legs to body. What do you know about
this?
Answer: This reflex should disappear between 1 and 4 months of age
73. The nurse is assessing a 75-year-old man. As the nurse beings the mental status portion of the
assessment, the nurse expects that this patient:
Answer: may take a little longer to respond, but his general knowledge and abilities should not
have declined
74. During an external genitalia examination of a woman, the nurse notices several lesions
around the vulva. The lesions are pink, moist, soft, and pointed papules. The patient states that
she is not aware of any problems in that area. The nurse recognizes that these lesions may be:
Answer: HPV or genital WARTs
75. A 40-year-old woman reports a change in mole size, accompanied by color changes, itching,
burning, and bleeding over the past month. She has a dark complexion and has no family history
of skin cancer, but she has had many blistering sunburns in the past. The nurse would:
Answer: Refer the patient because of the suspicion of melanoma on the basis of her symptoms.
76. Which of the following veins are responsible for most of the venous return in the arm?
Answer: Superficial veins
77. A man who has had gout for several years comes to the clinic with a "problem with my toe."
On examination, the nurse practitioner notes the presence of hard, painless nodules over the great
toe; one had burst open with a chalky discharge. This finding is known as:
Answer: Tophi are collections of sodium urate crystals resulting from chronic gout in and
around the joint that cause extreme swelling and joint deformity. They appear as hard, painless
nodules (tophi) over the metatarsophalangeal joint of the first toe and they sometimes burst with
a chalky discharge
78. The nurse is performing a well-child assessment on a 3-year-old child. The child's vital signs
are normal. Capillary refill time is 5 seconds. The nurse would:
Answer: consider this a delayed capillary refill time and investigate further
79. A 70-year-old patient is scheduled for open-heart surgery. The surgeon plans to use the great
saphenous vein for the coronary bypass grafts. The patient asks, "What happens to my circulation
when the veins are removed?" The nurse should reply:
Answer: "Because the deeper veins in your leg are in good condition, this vein can be removed
without harming your circulation."
80. When observing the vestibule, the nurse practitioner should be able to see the:
Answer: Urethral meatus and vaginal orifice
81. When the nurse is conducting sexual history from a male adolescent, which statement would
be most appropriate to use at the beginning of the interview?
Answer: Often boys your age have questions about sexual activity
82. The nurse practitioner is palpating the abdomen of a woman who is 35 weeks' pregnant and
notes that the fetal head is facing downward toward the pelvis. The nurse practitioner would
document this as:
Answer: Fetal presentation
83. Which of the following factors is most likely to affect the nutritional status of an 82-yearold
person?
Answer: Socioeconomic conditions frequently have the greatest effect on the nutritional status
of the aging adult; these factors should be closely evaluated. Physical limitations, income, and
social isolation are frequent problems and can obviously interfere with the acquisition of a
balanced diet.
84. When performing a genital examination on a 25-year-old man, the nurse notices deeply
pigmented, wrinkled scrotal skin with large sebaceous follicles. On the basis of this information
the nurse would:
Answer: consider this a normal finding and proceed with the examination.
85. Mrs. A has had arthritis for years and is starting to notice that her fingers are drifting to this
side. This is commonly referred to as:
Answer: ulnar deviation
86. A nurse is assessing a patient's risk of contracting a sexually transmitted infection (STI). An
appropriate question to ask would be:
Answer: "Do you use a condom with each episode of sexual intercourse?"
87. A 14-year-old boy who has been diagnosed with Osgood-Schlatter disease reports painful
swelling just below the knee for the past 5 months. Which response by the nurse practitioner is
appropriate?
Answer: "Your disease is due to repeated stress on the patellar tendon. It is usually self-limited,
and your symptoms should resolve with rest."
88. Which of the following is considered a normal and expected finding when the nurse
practitioner is performing a physical examination on a pregnant woman?
Answer: A palpable, full thyroid
89. A 54-year-old woman who has just completed menopause is in the clinic today for a yearly
physical examination. Which of these statements should the nurse include in patient education?
Answer: "A postmenopausal woman: should be aware that she is at increased risk for
dyspareunia because of decreased vaginal secretions."
90. The nurse practitioner has completed the musculoskeletal examination of a patient's knee and
has found a positive bulge sign. The nurse suspects:
Answer: Swelling from fluid in the suprapatellar pouch
91. Which of the following statements is true with regard to the history of a postmenopausal
woman?
Answer: The nurse should ask a postmenopausal woman if she ever has vaginal bleeding.
92. When performing a genital assessment on a middle-aged man, the nurse notices multiple soft,
moist, painless papules in the shape of cauliflower-like patches scattered across the shaft of the
penis. These lesions are characteristic of:
Answer: genital warts
93. Where did it first appear—on the nipple, the areola, or the surrounding skin? During a
physical examination, a 45-year-old woman states that she has had a crusty, itchy rash on her
breast for about 2 weeks. In trying to find the cause of the rash, which of these would be
important for the nurse to determine?
Answer: Where did it first appear—on the nipple, the areola, or the surrounding skin?
94. Which of the following statements is true regarding assessment of the ankle-brachial index
(ABI)?
Answer: ABI = highest ankle pressure divided by the highest brachial pressure
95. The patient is in her first trimester of pregnancy. She complains of feeling nauseated and has
vomited on occasion. She tells the nurse that she did not have this with her first pregnancy. She
asks the nurse, "What is causing this and when will it end?" How should the nurse respond?
Answer: The nausea is caused by elevated levels of progesterone and estrogen, and the nausea
should end once her body adjusts to the increased hormone levels
96. A professional tennis player comes in complaining of a sore elbow. You suspect that he has
tenderness at:
Answer: the medial and lateral epicondyle
97. During a bimanual examination, the nurse detects a solid tumor on the ovary that is heavy
and fixed, with a poorly defined mass. This finding is suggestive of:
Answer: ovarian cancer
98. When doing the history on a patient with a seizure disorder, the nurse assesses whether the
patient has an aura. Which of the following would be the best question for obtaining this
information?
Answer: Do you have any warning sign before your seizure starts
99. The nurse practitioner is assessing a 1-week-old infant and testing his muscle strength. The
nurse practitioner lifts the infant with hands under the axillae and notes that the infant starts to
"slip" between the hands. The nurse practitioner should:
Answer: Suspect that the infant may have weakness of the shoulder muscles
NSG 6020 WEEK 4 QUIZ
1. The aging process causes what normal physiological changes in the heart?
Answer: the heart valve thickens and becomes rigid, secondary to fibrosis and sclerosis
2. Dan G., a 65-year-old man, presents to your primary care office for the evaluation of chest
pain and left-sided shoulder pain. Pain begins after strenuous activity, including walking. Pain is
characterized as dull, aching; 8/10 during activity, otherwise 0/10. Began a few months ago,
intermittent, aggravated by exercise, and relieved by rest. Has occasional nausea. Pain is
retrosternal, radiating to left shoulder, definitely affects quality of life by limiting activity. Pain is
worse today; did not go away after he stopped walking. BP 120/80. Pulse 72 and regular. Normal
heart sounds, S1 and S2, no murmurs. Which of the following differential diagnoses would be
most likely
Answer: Coronary artery disease with angina pectoris
3. a common auscultatory finding in advanced CHF is
Answer: S3 gallop rhythm
4. Your 35-year-old female patient complains of feeling palpitations on occasion. The clinician
should recognize that palpitations are often a sign of
Answer: Anxiety, arrhythmias, or hyperthyroidism
5. The best way to diagnose structural heart disease/dysfunction non-invasively is
Answer: echocardiogram
6. which of the following is the most important question to ask during cardiovascular health
history
Answer: sudden death of a family member
7. a key symptom of ischemic heart disease is chest pain. However, angina equivalents may
include exertional dyspnea. Angina equivale stare important because
Answer: A and B (Women with ischemic heart disease/some patients may have no symptoms)
8. A 55 year old post-menopausal woman with a history of hypertension complains of jaw pain
on heavy exertion. There were no complaints of chest pain. Her ECG indicates normal sinus
rhythm without ST segment abnormalities. Your plan may include
Answer: exercise test
9. Jenny is a 24 year old graduate student that presents to the clinic today with complaints of
fever, mid sternal chest pain and generalized fatigue for the past two days. She denies any cough
or sputum production. She states that when she takes ibuprofen and rest that the chest pain does
seem to ease off. Upon examination the patient presents looking very ill. She is leaning forward
and states that this is the most comfortable position for her. Temp is 102, BP 100/70, heart rate is
120/min and regular. Upon auscultation a friction rub is audible. Her lungs sounds clear. With
these presenting symptoms your initial diagnosis would be
Answer: pericarditis
10. which symptom is more characteristic of non-cardiac chest pain
Answer: pain tends to occur with movement stretching or palpation
11. what is the most common valvular heart disease in the older adult aortic stenosis A survey
indicates that shoppers spend an average of 22 minutes with a standard deviation of 16 minutes
in your store and that these times are normally distributed. Find the probability that a randomly
selected shopper will spend less than 20 minutes in the store.
Answer: 0.45
12. The monthly utility bills in a city are normally distributed with a mean of $121 and a
standard deviation of $23. Find the probability that a randomly selected utility bill is between
$110 and $130.
Answer: 0.336
13. The blenders produced by a company have a normally distributed life span with a mean of
8.2 years and a standard deviation of 0.8 years. What warranty should be provided so that the
company is replacing at most 6% of their blenders sold?
Answer: 6.9 years
14. A puck company wants to sponsor the players with the 10% quickest goals in hockey games.
The times of first goals are normally distributed with a mean of 8.54 minutes and a standard
deviation of 4.91 minutes. How fast would a player need to make a goal to be sponsored by the
puck company?
Answer: 2.25 minutes
15. The times that customers spend in a book store are normally distributed with a mean of 39.5
minutes and a standard deviation of 15.9 minutes. A random sample of 30 customers has a mean
of 36.1 minutes or less. Would this outcome be considered unusual, so that the store should
reconsider its displays?
Answer: No, the probability of this outcome at 0.121, would be considered usual, so there is no
problem
16. The weights of ice cream cartons are normally distributed with a mean weight of 20 ounces
and a standard deviation of 0.3 ounces. You randomly select 40 cartons. What is the probability
that their mean weight is greater than 20.06 ounces?
Answer: 0.103
17. Jeff, 48 years old, presents to the clinic complaining of fleeting chest pain, fatigue,
palpitations, light-headedness, and shortness of breath. The pain comes and goes and is not
associated with activity or exertion. Food does not exacerbate or relieve the pain. The pain is
usually located under the left nipple. Jeff is concerned because his father has cardiac disease and
underwent a CABG at age 65. The ANP examines Jeff and hears a mid-systolic click at the 4th
ICS mid-clavicular area. The ANP knows that this is a hallmark sign of:
Answer: mitral valve prolapse
18. During auscultation of the chest, your exam reveals a loud grating sound at the lower
anterolateral lung fields, at full inspiration and early expiration. This finding is consistent with:
Answer: A and B (pneumonia/pleuritis)
19. A 75-year-old patient complains of pain and paraesthesia’s in the right foot that worsens with
exercise and is relieved by rest. On physical examination you note pallor of the right foot,
capillary refill of 4 seconds in the right foot, +1 dorsalis pedis pulse in the right foot, and +2
pulse in left foot. Which of the following is a likely cause of the signs and symptoms?
Answer: arterial insufficiency
20. Your patient complains of a feeling of heaviness in the lower legs daily. You note varicosities,
edema, and dusky color of both ankles and feet. Which of the following is the most likely cause
for these symptoms?
Answer: venous insufficiency
21. Your 54 year old patient, Mr. A, presents to your clinic with a 2 day history of severe
shoulder pain. On initial assessment you note that in addition to shoulder findings his blood
pressure on the 'good' arm is 162/100. You review his history and on his last visit his blood
pressure was 120/70. He has a medical history of sleep apnea and has used anabolic steroids
when body building as a younger adult. In addition to caring for Mr. A's chief complaint of
shoulder pain, you also
Answer: Schedule a follow up appointment after pain has subsided to take additional blood
pressure readings
22. You decide to order labs today to help with the diagnosis and management of hypertension in
Mr. A. Which of the following labs are indicated to assist in the medical management of Mr. A if
he meets the diagnostic criteria for hypertension?
Answer: fasting serum cholesterol panel
23. Lifestyle modifications reduce blood pressure, enhance antihypertensive medication efficacy,
and decrease cardiovascular risks. Which lifestyle change will decrease blood pressure the most?
Answer: weight reduction
24. Mr. A returns to your clinic and a diagnosis of hypertension is made. He is started on a
diuretic and counselled on lifestyle modifications including increasing activity and smoking
cessation. On his next visit you note that his blood pressure remains elevated. Before referring to
a specialist you should do all of the following except
Answer: Determine he has 'white coat' hypertension because his home readings are also elevated
25. Mr. A has many issues that seem to be interfering with his health outcomes. In order to
negotiate and formulate a patient-centred management plan you take the time to gather more
information. This can be started by asking the following question
Answer: What do you think caused your hypertension and how has it affected your life?
26. Which of the following is the most important question to ask during cardiovascular health
history?
Answer: Sudden death of a family member
27. A key symptom of ischemic heart disease is chest pain. However, angina equivalents may
include exertional dyspnea. Angina equivalents are important because
Answer: A & B only A. Women with ischemic heart disease many times do not present with
chest pain B. Some patients may have no symptoms or atypical symptoms. Diagnosis may only
be made at the time of an actual myocardial infarction
28. A 55-year-old post-menopausal woman with a history of hypertension complains of jaw pain
on heavy exertion. There were no complaints of chest pain. Her ECG indicates normal sinus
rhythm without ST segment abnormalities. Your plan may include
Answer: Exercise stress test
29. Which symptom is more characteristic of Non-Cardiac chest pain?
Answer: Pain tends to occur with movement, stretching or palpation
30. The aging process causes what normal physiological changes in the heart?
Answer: The heart valve thickens and becomes rigid, secondary to fibrosis and sclerosis
31. What is the most common valvular heart disease in the older adult?
Answer: Aortic stenosis
32. Jenny is a 24 year old graduate student that presents to the clinic today with complaints of
fever, midsternal chest pain and generalized fatigue for the past two days. She denies any cough
or sputum production. She states that when she takes Ibuprofen and rest that the chest pain does
seem to ease off. Upon examination the patient presents looking very ill. She is leaning forward
and states that this is the most comfortable position for her. Temp is 102. BP= 100/70. Heart rate
is 120/min and regular. Upon auscultation a friction rub is audible. Her lung sounds are clear.
With these presenting symptoms your initial diagnosis would be
Answer: Pericarditis
33. A common auscultatory finding in advanced CHF is
Answer: S3 gallop rhythm
34. Mr. A returns to your clinic and a diagnosis of hypertension is made. He is started on a
diuretic and counselled on lifestyle modifications including increasing activity and smoking
cessation. On his next visit you note that his blood pressure remains elevated. Before referring to
a specialist you should do all of the following except
Answer: Determine he has 'white coat' hypertension because his home readings are also elevated
35. Jeff, 48 years old, presents to the clinic complaining of fleeting chest pain, fatigue,
palpitations, light-headedness, and shortness of breath. The pain comes and goes and is not
associated with activity or exertion. Food does not exacerbate or relieve the pain. The pain is
usually located under the left nipple. Jeff is concerned because his father has cardiac disease and
underwent a CABG at age 65. The ANP examines Jeff and hears a mid-systolic click at the 4th
ICS mid-clavicular area. The ANP knows that this is a hallmark sign of:
Answer: Mitral valve prolapsed
NSG 6020 WEEK 5
1. On the production line the company finds that 85.6% of products are made correctly. You are
responsible for quality control and take batches of 30 products from the line and test them. What
number of the 30 being incorrectly made would cause you to shut down production?
Answer: Less than 23
2. The probability of someone ordering the daily special is 71%. If the restaurant expected 65
people for lunch, how many would you expect to order the daily special?
Answer: 46
3. Sixty-five percent of employees make judgements about their co-workers based on the
cleanliness of their desk. You randomly select 8 employees and ask them if they judge coworkers based on this criterion. The random variable is the number of employees who judge their
co-workers by cleanliness. Which outcomes of this binomial distribution would be considered
unusual?
Answer: 0, 1, 2, 8
4. Sixty-five percent of products come off the line ready to ship to distributors. Your quality
control department selects 12 products randomly from the line each hour. Looking at the
binomial distribution, if fewer than how many are within specifications would require that the
production line be shut down (unusual) and repaired?
Answer: Fewer than 5
5. Out of each 100 products, 93 are ready for purchase by customers. If you selected 20 products,
what would be the expected (mean) number that would be ready for purchase by customers?
Answer: 18 – 19
6. If you created the probability distribution for these data, what would be the probability of 4049?
Answer: 0.165
7. Find the mean of this variable.
Answer:16.80
8. Find the variance of this variable.
Answer: 2.08
9. Find the standard deviation of this variable.
Answer: 1.44
10. Fifty-four percent of US teens have heard of a fax machine. You randomly select 12 US
teens. Find the probability that the number of these selected teens that have heard of a fax
machine is exactly six (first answer listed below). Find the probability that the number is more
than 8 (second answer listed below).
Answer: 0.217, 0.120
11. Ten rugby balls are randomly selected from the production line to see if their shape is correct.
Over time, the company has found that 92.4% of all their rugby balls have the correct shape. If
exactly 7 of the 10 have the right shape, should the company stop the production line?
Answer: Yes, as the probability of seven having the correct shape is unusual
12. A bottle of water is supposed to have 12 ounces. The bottling company has determined that
98% of bottles have the correct amount. Which of the following describes a binomial experiment
that would determine the probability that a case of 36 bottles has all bottles properly filled?
Answer: n=36, p=0.98, x=36
13. Sixty-seven percent of adults have looked at their credit score in the past six months. If you
select 31 customers, what is the probability that at least 25 of them have looked at their score in
the past six months?
Answer: 0.073
14. One out of every 92 tax returns that a tax auditor examines requires an audit. If 50 returns are
selected at random, what is the probability that less than 5 will need an audit?
Answer: 0.9998
15. Thirty-eight percent of consumers prefer to purchase electronics online. You randomly select
16 consumers. Find the probability that the number who prefer to purchase electronics online is
at most 6.
Answer: 0.593
16. The speed of cars on a stretch of road is normally distributed with an average 48miles per
hour with a standard deviation of 5.9 miles per hour. What is the probability that a randomly
selected car is violating the speed limit of 50 miles per hour?
Answer: 0.37
17. A survey indicates that shoppers spend an average of 22 minutes with a standard deviation of
8 minutes in your store and that these times are normally distributed. Find the probability that a
randomly selected shopper will spend less than 20 minutes in the store.
Answer: 0.40
18. The monthly utility bills in a city are normally distributed with a mean of $121 and a
standard deviation of $23. Find the probability that a randomly selected utility bill is between
$115 and $130.
Answer: 0.255
19. A restaurant serves hot chocolate that has a mean temperature of 175 degrees with a standard
deviation of 6.2 degrees. Find the probability that a randomly selected cup of hot chocolate
would have a temperature of less than 164 degrees. Would this outcome warrant a replacement
cup (meaning that it would be unusual)?
Answer: Probability of 0.04 and would warrant a refund
20. The yearly amounts of carbon emissions from cars in Belgium are normally distributed with
a mean of 13.9 gigagrams per year and a standard deviation of 9.2 gigagrams per year. Find the
probability that the amount of carbon emissions from cars in Belgium for a randomly selected
year are between 12.8 gigagrams and 14.0 gigagrams per year.
Answer:0.052
21. On average, the parts from a supplier have a mean of 97.5 inches and a standard deviation of
6.1 inches. Find the probability that a randomly selected part from this supplier will have a value
between 87.5 and 107.5 inches. Is this consistent with the Empirical Rule of 68%-95%-99.7%?
Answer: Probability is 0.90, which is inconsistent with the Empirical Rule
22. A process is normally distributed with a mean of 104 rotations per minute and a standard
deviation of 8.2 rotations per minute. If a randomly selected minute has 80 rotations per minute,
would the process be considered in control or out of control?
Answer: In control as this one data point is not more than three standard deviations from the
mean
23. The soup produced by a company has a salt level that is normally distributed with a mean of
5.4 grams and a standard deviation of 0.3 grams. The company takes readings of every 10th bar
off the production line. The reading points are 5.8, 5.9, 4.9, 5.7, 5.0, 4.9, 5.5, 5.1, 5.7, 6.1. Is the
process in control or out of control and why?
Answer: It is in control as only one data point is more than 2 standard deviations from the mean
24. The blenders produced by a company have a normally distributed life span with a mean of
8.2 years and a standard deviation of 1.3 years. What warranty should be provided so that the
company is replacing at most 6% of their blenders sold?
Answer: 6.2 years
25. A puck company wants to sponsor the players with the 20% quickest goals in hockey games.
The times of first goals are normally distributed with a mean of 8.54 minutes and a standard
deviation of 4.91 minutes. How fast would a player need to make a goal to be sponsored by the
puck company?
Answer: 4.41 minutes
26. A stock's price fluctuations are approximately normally distributed with a mean of $104.50
and a standard deviation of $23.62. You decide to purchase whenever the price reaches its lowest
10% of values. What is the most you would be willing to pay for the stock?
Answer: $74.23
27. The times that customers spend in a book store are normally distributed with a mean of 39.5
minutes and a standard deviation of 15.9 minutes. A random sample of 60 customers has a mean
of 36.1 minutes or less. Would this outcome be considered unusual, so that the store should
reconsider its displays?
Answer: Yes, the probability of this outcome at 0.049, would be considered unusual, so the
display should be redone
28. The weights of ice cream cartons are normally distributed with a mean weight of 20 ounces
and a standard deviation of 0.5 ounces. You randomly select 25 cartons. What is the probability
that their mean weight is greater than 20.06 ounces?
Answer: 0.274
29. Recent test scores on the Law School Admission Test (LSAT) are normally distributed with a
mean of 162.4 and a standard deviation of 15.9. What is the probability that the mean of 8
randomly selected scores is less than 161?
Answer: 0.402
30. The mean annual salary for intermediate level executives is about $74000 per year with a
standard deviation of $2500. A random sample of 36 intermediate level executives is selected.
What is the probability that the mean annual salary of the sample is between $71000 and
$74000?
Answer: 0.500
31. If you created the probability distribution for these data, what would be the probability of 3039?
Answer: 0.189
32. Find the mean of this variable.
Answer: 17.65
33. Find the variance of this variable.
Answer: 1.99
34. Find the standard deviation of this variable.
Answer: 1.41
35. Forty-nine percent of US teens have heard of a fax machine. You randomly select 12 US
teens. Find the probability that the number of these selected teens that have heard of a fax
machine is exactly six (first answer listed below). Find the probability that the number is more
than 8 (second answer listed below).
Answer: 0.225, 0.064
36. A bottle of water is supposed to have 20 ounces. The bottling company has determined that
98% of bottles have the correct amount. Which of the following describes a binomial experiment
that would determine the probability that a case of 24 bottles has all bottles properly filled?
Answer: n=24,
p=0.98, x=24
37. On the production line the company finds that 99.7% of products are made correctly. You are
responsible for quality control and take batches of 30 products from the line and test them. What
number of the 30 being incorrectly made would cause you to shut down production?
Answer: Less than 29
38. The probability of someone ordering the daily special is 52%. If the restaurant expected 65
people for lunch, how many would you expect to order the daily special?
Answer:34
39. Fifty-seven percent of employees make judgements about their co-workers based on the
cleanliness of their desk. You randomly select 8 employees and ask them if they judge coworkers based on this criterion. The random variable is the number of employees who judge their
co-workers by cleanliness. Which outcomes of this binomial distribution would be considered
unusual?
Answer: 0, 1, 8
40. Seventy-nine percent of products come off the line ready to ship to distributors. Your quality
control department selects 12 products randomly from the line each hour. Looking at the
binomial distribution, if fewer than how many are within specifications would require that the
production line be shut down (unusual) and repaired?
Answer: Fewer than 7
41. Sixty percent of adults have looked at their credit score in the past six months. If you select
31 customers, what is the probability that at least 25 of them have looked at their score in the
past six months?
Answer: 0.013
42. The speed of cars on a stretch of road is normally distributed with an average 42 miles per
hour with a standard deviation of 5.9 miles per hour. What is the probability that a randomly
selected car is violating the speed limit of 50 miles per hour?
Answer: 0.09
43. A survey indicates that shoppers spend an average of 26 minutes with a standard deviation of
8 minutes in your store and that these times are normally distributed. Find the probability that a
randomly selected shopper will spend less than 20 minutes in the store.
Answer: 0.23
44. The monthly utility bills in a city are normally distributed with a mean of $128 and a
standard deviation of $23. Find the probability that a randomly selected utility bill is between
$110 and $130.
Answer: 0.318
45. A restaurant serves hot chocolate that has a mean temperature of 175 degrees with a standard
deviation of 8.1 degrees. Find the probability that a randomly selected cup of hot chocolate
would have a temperature of less than 164 degrees. Would this outcome warrant a replacement
cup (meaning that it would be unusual)?
Answer: Probability of 0.09 and would not warrant a refund
46. The yearly amounts of carbon emissions from cars in Belgium are normally distributed with
a mean of 13.9 gigagrams per year and a standard deviation of 3.5 gigagrams per year. Find the
probability that the amount of carbon emissions from cars in Belgium for a randomly selected
year are between 11.5 gigagrams and 14.0 gigagrams per year.
Answer: 0.265
47. On average, the parts from a supplier have a mean of 97.5 inches and a standard deviation of
12.2 inches. Find the probability that a randomly selected part from this supplier will have a
value between 87.5 and 107.5 inches. Is this consistent with the Empirical Rule of 68%-95%99.7%?
Answer: Probability is 0.59, which is inconsistent with the Empirical Rule
48. A process is normally distributed with a mean of 104 rotations per minute and a standard
deviation of 8.2 rotations per minute. If a randomly selected minute has 118 rotations per minute,
would the process be considered in control or out of control?
Answer: In control as this one data point is not more than three standard deviations from the
mean
49. The soup produced by a company has a salt level that is normally distributed with a mean of
5.4 grams and a standard deviation of 0.3 grams. The company takes readings of every 10th bar
off the production line. The reading points are 5.8, 5.9, 4.9, 5.2, 5.0, 4.9, 6.2, 5.1, 5.3, 6.7. Is the
process in control or out of control and why?
Answer: It is out of control as one of these data points is more than 3 standard deviations from
the mean
50. The blenders produced by a company have a normally distributed life span with a mean of
8.2 years and a standard deviation of 1.3 years. What warranty should be provided so that the
company is replacing at most 10% of their blenders sold?
Answer: 6.5 years
51. A puck company wants to sponsor the players with the 10% quickest goals in hockey games.
The times of first goals are normally distributed with a mean of 12.56 minutes and a standard
deviation of 4.91 minutes. How fast would a player need to make a goal to be sponsored by the
puck company?
Answer: 6.27 minutes
52. A stock's price fluctuations are approximately normally distributed with a mean of $104.50
and a standard deviation of $20.88. You decide to purchase whenever the price reaches its lowest
20% of values. What is the most you would be willing to pay for the stock?
Answer: $86.93
53. The times that customers spend in a book store are normally distributed with a mean of 39.5
minutes and a standard deviation of 15.9 minutes. A random sample of 25 customers has a mean
of 36.1 minutes or less. Would this outcome be considered unusual, so that the store should
reconsider its displays?
Answer: No, the probability of this outcome at 0.142, would be considered usual, so there is no
problem
54. The weights of ice cream cartons are normally distributed with a mean weight of 20.1 ounces
and a standard deviation of 0.3 ounces. You randomly select 25 cartons. What is the probability
that their mean weight is greater than 20.06 ounces?
Answer: 0.748
55. If you created the probability distribution for these data, what would be the probability of 5059?
Answer: 0.152
56. Find the standard deviation of this variable.
Answer: 1.40
57. Forty-three percent of US teens have heard of a fax machine. You randomly select 12 US
teens. Find the probability that the number of these selected teens that have heard of a fax
machine is exactly six (first answer listed below). Find the probability that the number is more
than 8 (second answer listed below).
Answer: 0.200, 0.026
58. Ten rugby balls are randomly selected from the production line to see if their shape is correct.
Over time, the company has found that 89.4% of all their rugby balls have the correct shape. If
exactly 6 of the 10 have the right shape, should the company stop the production line?
Answer: Yes, as the probability of six having the correct shape is unusual
59. A bottle of water is supposed to have 20 ounces. The bottling company has determined that
96% of bottles have the correct amount. Which of the following describes a binomial experiment
that would determine the probability that a case of 36 bottles has all bottles properly filled?
Answer: n=36,
p=0.96, x=36
60. Forty-seven percent of employees make judgements about their co-workers based on the
cleanliness of their desk. You randomly select 8 employees and ask them if they judge coworkers based on this criterion. The random variable is the number of employees who judge their
co-workers by cleanliness. Which outcomes of this binomial distribution would be considered
unusual?
Answer: 0, 1, 7, 8
61. One out of every 92 tax returns that a tax auditor examines requires an audit. If 70 returns are
selected at random, what is the probability that less than 5 will need an audit?
Answer: 0.9990
62. Thirty-eight percent of consumers prefer to purchase electronics online. You randomly select
16 consumers. Find the probability that the number who prefer to purchase electronics online is
at most 4.
Answer: 0.211
NSG 6020 WEEK 6
1. What is a "scheme"?
Answer: hat are used repeatedly in response to the environment ( strategies)
2. What is the difference between "assimilation" and "accommodation"?
Answer: • Assimilation = applying an existing scheme Accommodation = 1) modifying an
existing scheme or 2) forming a new scheme
3. Explain the process of "equilibration" (use the terms "equilibrium" and "disequilibrium").
Answer: Equilibration = progression toward increasingly complex forms of thought by cycling
between equilibrium and disequilibrium
4. What type of methodology did Piaget use in his research?
Answer: Rigorous methods, but not experimental (naturalistic observation, psychometrics,
clinical interview)
5. Social learning theory emerged from which -ism
Answer: behaviourism
6. What are possible reinforcers for social learning theory?
Answer: Models, third person, imitated behavior, consequences (vicarious punishment and
reinforcement)
7. Who was a key leader in the development of the social cognitive theory?
Answer: Albert Bandura
8. Principle One (SCT)
Answer: People can learn by observing others' behaviours and the consequences that result
9. Principle Two (SCT)
Answer: Learning can occur without a change in behavior
10. Principle Three (SCT)
Answer: Cognition plays important roles in learning
11. Principle Four (SCT)
Answer: People can have considerable control over their actions and environments
12. Three elements of reciprocal causation
Answer: Person, Behavior (observable actions), and Environment (stimuli from the outside
world)
13. Modelling
Answer: Learning by observing a behavior
14. Types of models
Answer: live models, symbolic models, and verbal instructions
15. what are the characteristics of effective model
Answer: Competency, Prestige and power, stereotypical behavior, and relevance to observer
16. What the conditions for effective modelling?
Answer: Attention, Retention, Motor reproduction, and motivation
17. Social Learning Theory can best be characterized as being concerned with learning:
Answer: through observations of others
18. Which one of the following is an example of vicarious reinforcement?
1) Bill knows that he will get a higher grade if he turns in a research paper that is typed rather
than handwritten, but he turns in a handwritten paper anyway
2) Alice notices that her friend Ellen gets extra attention from the teacher when she acts helpless.
Alice begins to act helpless as well.
3) Connie sees her friend Maria scolded for chewing gum in class. She quickly takes her own
gum out of her mouth.
Answer: 2) Alice notices that her friend Ellen gets extra attention from the teacher when she acts
helpless. Alice begins to act helpless as well.
19. The person who performs a behavior that serves as an example is called a ________.
Answer: model
20. In Bandura's Bobo doll study, when the children who watched the aggressive model were
placed in a room with the doll and other toys, they ________.
Answer: kicked and threw the doll
21. Jessy was speeding on his motorcycle when he saw another person pulled over by the police.
This made Jessy slow down because he was reminded through watching another person what the
punishment of his behavior would be. What phenomenon does this example illustrate?
1) vicarious punishment
2) operant conditioning
3) vicarious reinforcement
4) classical conditioning
Answer: 1) vicarious punishment
22. Cory's dad hides his keys under her blanket. Cory quickly retrieves and starts playing with
them. What Piagetian stage has Cory mastered?
1) pre-operational
2) formal operational
3) concrete operational
4) sensorimotor
Answer: 4) sensorimotor
23. Which one of the following statements best describes Piaget's view of how children acquire
knowledge about the world?
1) Children are naturally disposed to think about their environment in particular ways; in a sense,
some basic knowledge about the world is "pre-wired."
2) Children repeatedly parrot their parents' and teachers' beliefs, eventually internalizing these
beliefs as their own "knowledge."
3) Children actively construct their own view of the world from their experiences with the
environment.
Answer: 3) Children actively construct their own view of the world from their experiences with
the environment.
24. Piaget has described four stages of cognitive development. Which one of the following is an
accurate statement about Piaget's view of these stages?
1) With each successive stage, schemata become more simplified.
2) Each stage involves forms of thought qualitatively different from those of other stages.
3) It is possible for children to skip from pre operations to formal operations, but only a small
minority of children actually do so.
Answer: 2) Each stage involves forms of thought qualitatively different from those of other
stages.
25. If we look at cognitive development from Piaget's perspective, we would expect a child in
the concrete operations stage to have the greatest difficulty with which
1) If you have 8 Macintosh apples and 2 Golden Delicious apples, then do you have more
Macintoshes or more apples?
2) An apple pie is cut into 4 pieces. A blueberry pie of the same size is cut into 12 pieces. How
many pieces of blueberry pie do you need to have the same amount as 3 pieces of the apple pie?
3) If we have one row of blueberries spread like so: and another row of blueberries spread like
so: then does one row have more blueberries than the other?
4) How are an apple and a blueberry alike? one of the following questions?
Answer: 1) If you have 8 Macintosh apples and 2 Golden Delicious apples, then do you have
more Macintoshes or more apples?
26. During the preoperational stage, children struggle with ________ problems as they focus on
one dimension and ignore changes in other dimensions. For example, laying out five candies
close together versus spread out will likely lead a 2- to 7-year-old to state that the second layout
has more candy (even though the actual number of candies is the same).
Answer: conservation
27. How does social cognitive theory differ from cognitivist theory?
Answer: Cognitivist = how do cognitive factors influence learning and behavior SCT = How do
environmental and cognitive factors interact to influence learning and behavior
28. What is vicarious reinforcement?
Answer: Reinforcement which is received indirectly by observing another person who is being
reinforced
29. How are a person, their behavior, and the environment related within the idea of reciprocal
causation?
Answer: More specifically, learning and long-term development occur through the interaction of
these three sets of variables
30. Why did social cognitive theory emerge from social learning theory?
Answer: The inability to explain many social learning phenomena with the ideas and principles
of behaviourism.
31. what is Piaget’s idea of constructivism?
Answer: children construct their own knowledge through interaction with the world
32. What was Piaget’s goal?
Answer: to examine children's thought processes by observing how they respond and articulate
their reasoning
33. Schemes (Piaget)
Answer: organized groups of similar actions or thoughts that are used repeatedly in response to
the environment ( strategies)
34. Operations (Piaget)
Answer: cognitive structures constructed from the integration of multiple schemes that govern
logical reasoning
35. How does learning occur (Piaget)?
Answer: Occurs through interaction with the physical environment and other people
(assimilation and accommodation)
36. Assimilation
Answer: applying an existing scheme
37. Accommodation
Answer: modifying an existing scheme or forming a new scheme.
38. Equilibration (Piaget)
Answer: progression toward increasingly complex forms of thought and occurs through repeated
cycling between two states
39. Equilibrium (Piaget)
Answer: children can comfortably interpret and respond to new events using existing schemes
40. Disequilibrium (Piaget)
Answer: "mental discomfort" caused by situations for which existing schemes are inadequate
41. Piaget's four stages of development
Answer: • Sensorimotor (age 0-2)
• Preoperational (age 2-7)
• concrete operations (age 7-11)
• formal operations (age 11 and on)
42. sensorimotor stage
Answer: in Piaget's theory, the stage (from birth to nearly 2 years of age) during which infants
know the world mostly in terms of their sensory impressions and motor activities and acquire
knowledge of cause-effect and develop object permanence
43. preoperational stage
Answer: in Piaget's theory, the stage (from about 2 to 6 or 7 years of age) where they develop
egocentrism, increase complexity of thought, can think about past/future, and thinking depends
on perception more than logic
44. concrete operational stage
Answer: in Piaget's theory, the stage of cognitive development (from about 6 or 7 to 11 years of
age) during which children gain the mental operations that enable them to think logically about
concrete events and take different perspectives and have trouble with abstract reasoning.
45. Formal operations stage
Answer: Piaget's last stage of cognitive development, in which the adolescent becomes capable
of abstract thinking, can test hypothesis, have metacognition
46. Piagets Legacy
Answer: 1) Learning through doing
2) Disequilibrium spurs acquisition of new knowledge
3) Learning from peers
4) Complexity of new learning depends on existing knowledge
5) Effectiveness of clinical interview method
6) Stages ( gradual trends) are pedagogically useful
47. Neo-Piagetian
Answer: Combination of Piagetian and cognitivist ideas
48. Key Ideas in Neo-Piagetian Theories
Answer: 1) Cognitive development depends on brain maturation
2) Learning occurs intentionally and unintentionally
3) Acquisition of knowledge is domain-specific
49. Explain the role of neurological maturation in Piaget's theory of cognitive development.
Answer: Children think in qualitatively different ways at different age levels because of
genetically controlled developmental changes in the brain (i.e. neurological maturation)
50. Name one ability that children acquire when they reach the preoperational stage (age 2-7).
Answer: Development of language, increase in complexity of thought, ability to think about past
and future events, among others
NSG 6020 WEEK 7
1. Patient needs to be told to stop compressing her nipple.
Answer: benign breast abnormality
2. fixed flattening of the nipple suggest (recent or fixed flattening or depression of the nipple)
may also be broadened and thickened
Answer: nipple retraction
3. internal malignancy associated with polycystic ovarian syndrome, consisting of acne,
hirsutism, obesity, irregular periods, infertility, ovarian cys, and eterm-6arly onset type II
diabetes
Answer: acanthosis nigricans
4. a 17 year old female patient complains of amenorrhea for the last 8 months, weight gain,
excessive hair growth on the arms and chest, and development of acne. Pelvic examination is
normal. The clinician should recognize these are signs of
Answer: hormonal imbalances
5. uncommon form of breast cancer that starts with a scaly, eczema-like lesion that may weep,
crust , or erode
Answer: pagets disease
6. which of the following is true of human papilloma virus infection
Answer: it commonly resolves spontaneously in one to two years
7. Jean has just given birth 6 months ago and is breast-feeding her child. She has not had a period
since giving birth. What does this most likely represent?
Answer: Secondary amenorrhea
8. A 24-year-old travel agent comes to your clinic, complaining of pain and swelling in her
vulvar area. She states that 2 days earlier she could feel a small tender spot on the left side of her
vagina but now it is larger and extremely tender. Her last period was 1 year ago and she is
sexually active. She uses the Depo-Provera shot for contraception. She denies any nausea,
vomiting, constipation, diarrhoea, pain with urination, or fever. Her past medical history is
significant for ankle surgery. Her mother is healthy and her father has type 2 diabetes. On
examination she appears her stated age and is standing up. She states she cannot sit down
without excruciating pain. Her blood pressure, temperature, and pulse are unremarkable. On
visualization of her perineum, a large, red, tense swelling is seen to the left of her introitus.
Palpation of the mass causes a great deal of pain. What disorder of the vulva is most likely
Answer: Bartholin's gland infection
9. A 48-year-old high school librarian comes to your clinic, complaining of 1 week of heavy
discharge causing severe itching. She is not presently sexually active and has had no burning
with urination. The symptoms started several days after her last period. She just finished a course
of antibiotics for a sinus infection. Her past medical history consists of type 2 diabetes and high
blood pressure. She is widowed and has three children. She denies tobacco, alcohol, or drug use.
Her mother has high blood pressure and her father died of diabetes complications. On
examination you see a healthy-appearing woman. Her blood pressure is 130/80 and her pulse is
70. Her head, eyes, ears, nose, throat, cardiac, lung, and abdominal examinations are
unremarkable. Palpation of the inguinal lymph nodes is unremarkable. On visualization of the
vulva, a thick, white, curdy discharge is seen at the introitus. On speculum examination their
Answer: Candida vaginitis
10. Which of the following represents metrorrhagia?
Answer: bleeding between periods
11. A 42-year-old realtor comes to your clinic, complaining of "growths" in her vulvar area. She
is currently undergoing a divorce and is convinced she has a sexually transmitted disease. She
denies any vaginal discharge or pain with urination. She has had no fever, malaise, or night
sweats. Her past medical history consists of depression and hypothyroidism. She has had two
spontaneous vaginal deliveries and one caesarean section. She has had no other surgeries. She
denies smoking or drug use. She has two to three drinks weekly. Her mother also has
hypothyroidism and her father has high blood pressure and hypercholesterolemia. On
examination you see a woman who is anxious but appears otherwise healthy. Her blood pressure,
pulse, and temperature are unremarkable. On visualization of the perineum you see two 2- to 3mm, round, yellow nodules on the left labia. On palpation they are nontender and quite firm.
What diagnosis b
Answer: epidermoid cyst
12. examination of a female patients right breast reveals a retraction of the nipple and areola.
This finding is consistent with? (nipple discharge is usually unilateral and can be clear or bloody)
can have fixed lump and present as chronic rash on the breast
Answer: breast cancer
13. Abby is a newly married woman who is unable to have intercourse because of vaginismus.
Which of the following is true?
Answer: Psychosocial reasons may cause this condition.
14. It has been shown to reduce mortality from breast cancer
Answer: a high proportion of breast masses are detected by breast self-examination
15. one breast larger than the other
Answer: benign breast examination
16. If diagnosed with gonorrhoea should be treated from the following infection
Answer: chlamydia
17. A 34-year-old married daycare worker comes to your office, complaining of severe pelvic
pain for the last 6 hours. She states that the pain was at first cramp-like but is now sharp. Nothing
makes the pain better or worse. She has had no vaginal bleeding or discharge. She has had no
pain with urination. She has had some nausea for the last few days but denies vomiting,
constipation, or diarrhoea. She states she feels so bad that when she stands up, she has fainted.
Her past medical history consists of two prior caesarean sections and an appendectomy. She is
married and has two children. She denies any tobacco, alcohol, or drug use. Her parents are both
healthy. On examination you find a pale young woman who is obviously in a great deal of pain.
She is lying on her right side with her eyes closed. Her blood pressure is 90/60 and her pulse is
110.
She is afebrile. She has bowel sounds and her abdomen is soft. The speculum
Answer: tubal pregnancy
18. A 22-year-old architecture major comes to your office, complaining of severe burning with
urination, a fever of 101 degrees, and aching all over. She denies any upper respiratory,
gastrointestinal, cardiac, or pulmonary symptoms. Her past medical history consists of severe
acne. She is currently on an oral contraceptive. She has had no pregnancies or surgeries. She
reports one new partner within the last month. She does not smoke but does drink occasionally.
Her parents are both in good health. On examination you see a young woman appearing slightly
ill. Her temperature is 100.3 and her pulse and blood pressure are unremarkable. Her head, ears,
eyes, nose, throat, cardiac, pulmonary, and abdominal examinations are unremarkable. Palpation
of the inguinal nodes shows lymphadenopathy bilaterally. On visualization of the perineum there
are more than 10 shallow ulcers along each side of the vulva. Speculum and bimanual ex
Answer: genital herpes
19. A 30-year-old paralegal analyst comes to your clinic, complaining of a bad-smelling vaginal
discharge with some mild itching, present for about 3 weeks. She tried douching but it did not
help. She has had no pain with urination or with sexual intercourse. She has noticed the smell
increased after intercourse and during her period last week. She denies any upper respiratory,
gastrointestinal, cardiac, or pulmonary symptoms. Her past medical history consists of one
spontaneous vaginal delivery. She is married and has one child. She denies tobacco, alcohol, or
drug use. Her mother has high blood pressure and her father died from a heart disease. On
examination she appears healthy and has unremarkable vital signs. On examination of the
perineum there are no lesions noted. On palpation of the inguinal nodes there is no
lymphadenopathy. On speculum examination a thin Gray-white discharge is seen in the vault.
The pH of the
Answer: bacterial vaginosis
20. A 55-year-old married homemaker comes to your clinic, complaining of 6 months of vaginal
itching and discomfort with intercourse. She has not had a discharge and has had no pain with
urination. She has not had a period in over 2 years. She has no other symptoms. Her past medical
history consists of removal of her gallbladder. She denies use of tobacco, alcohol, and illegal
drugs. Her mother has breast cancer and her father has coronary artery disease, high blood
pressure, and Alzheimer's disease. On examination she appears healthy and has unremarkable
vital signs. There is no lymphadenopathy with palpation of the inguinal nodes. Visualization of
the vulva shows dry skin but no lesions or masses. The labia are somewhat smaller than usual.
Speculum examination reveals scant discharge and the vaginal walls are red, dry, and bleed
easily. Bimanual examination is unremarkable. The KOH whiff test produces no unusual Odor a
Answer: atrophic vaginitis
NSG 6020 WEEK 8
1. Which of the following symptoms would most likely be related to an inguinal hernia?
Answer: Pain increases with straining
2. Which of the following is not considered a risk for prostate cancer?
Answer: History of BPH
3. Which of the following information regarding testicular cancer is false?
Answer: Testicular cancer is five times more common in African American males compared to
Caucasian males.
4. Which of the following symptoms is not indicated for a prompt clinical examination?
Answer: One testicle "hanging" lower than the other
5. While examining the penis, the clinician notes a tight prepuce that cannot be retracted over the
glans. Which if the following is the correct diagnosis?
Answer: Phimosis
6. 21-year-old male reports a two-day history of profuse yellow penis discharge. The final
diagnosis revealed gonorrhoea. Which of the following infections would also be treated in light
of this diagnosis?
Answer: Chlamydia
7. Which of the following differential diagnosis must be considered in a patient with tender,
painful scrotal swelling?
Answer: Epididymitis, orchitis, torsion of the testicle, hydrocele, inguinal hernia, testicular
tumor, trauma or injury, and abscess.
8. While examining a male for scrotal swelling, the clinician notes a red glow upon shinning a
light behind the scrotum. Which of the following is the most likely diagnosis?
Answer: Hydrocele
9. An abnormal dilation of the spermatic veins that drain the testicle is considered which of the
following?
Answer: Varicocele
10. A 22-year-old male presented with a 3-day history of painful lesions on the shaft of his penis.
Symptoms include fever and malaise. On examination, the findings reveal a cluster of vesicular
lesions along the ventral aspect of the penis. Positive lymphadenopathy noted in inguinal area.
Which of the following is the most likely diagnosis?
Answer: Herpes simplex
11. Objective data related to the male GU system would include which of the following?
Answer: Firm enlarged prostate
12. Bulging noted in the inguinal area in a male patient is most likely to be which of the
following?
Answer: Hernia
13. Self-testicular exams should be performed how frequently in males less than age 40?
Answer: Once a month
14. Objective findings in the older male patient may include which of the following?
Answer: Prostatic Hypertrophy
15. Which of the following is a symptom of benign prostatic hyperplasia?
Answer: Nocturia
16. Which of the following symptoms would not be associated with acute testicular torsion?
Answer: Unilateral painless swelling of the testicle
17. Which of the following is not a common sign of direct inguinal hernias?
Answer: Often found in children
18. Which of the following is not a common symptom in males with BPH?
Answer: Absence of nocturia
19. Which of the following is not associated with acute bacterial prostatitis?
Answer: Chlamydia and gonorrhoea may be the cause in males older than 35 years
20. Which of the following is not associated with chronic bacterial prostatitis?
Answer: Presence of fever
21. Which of the following is inaccurate in regard to an undescended testicle (cryptorchidism)?
Answer: Approximately 2/3 of cases of cryptorchidism, the testicle will descend by age 1 year
22. Which of the following risk factors is not associated with epididymitis?
Answer: Painless swelling of the scrotum
23. Which of the following is not considered a red flag in the male reproductive system?
Answer: Erection lasting longer than 15 minutes after cessation of sexual activity
24. General history involving the male reproductive history should include which of the
following?
Answer: General history involving the male reproductive history should include sexual history,
past medical history, symptoms, family history, fertility history, medications, and lifestyle
factors.
NSG 6020 WEEK 9
1. Ms. fletcher comes to your office with unilateral pain during chewing, which is chronic. She
does not have facial tenderness or tenderness of the scalp. Which of the following is the most
likely cause of her pain
Answer: temporomandibular joint syndrome
2. a man's wife is upset because when sh hugs him with her hands on his left shoulder blade. "it
feels creepy." This came on gradually after a recent severe left-sided rotator cuff tear. How long
does it usually take to develop muscular atrophy with increased prominence of the scapular spine
following a rotator cuff tear
Answer: 2-3 weeks
3. Phil comes to your office with left "shoulder pain." You find that the pain is markedly worse
when his left arm is drawn across his chest (adduction). Which of the following would you
suspect?
Answer: acromioclavicular joint movement
4. Two weeks ago, Mary started a job which requires carrying 40-pound buckets. She presents
with elbow pain worse on the right. On examination, it hurts her elbows to dorsiflex her hands
against resistance when her palms face the floor. What condition does she have?
Answer: lateral epicondylitis (tennis elbow)
5. a high school football player injured his wrist in a game. He is tender between the two tendons
at the base of the thumb. Which of the following should be considered
Answer: scaphoid fracture
6. Ms. Fletcher complains of numbness of her right hand. On examination, sensation of the volar
aspect of the web of the thumb and index finger and the pulp of the middle finger are normal. the
pulp of the index finger has decreased sensation. which of the following is affected
Answer: median nerve
7. a 50-year-old woman presents with "left hip pain" of several weeks duration. There is marked
tenderness when you press over her proximal lateral thigh. What do you think she has
Answer: trochanteric bursitis
8. Sarah presents with left lateral knee pain and has some locking in full extension. There is
tenderness over the medial joint line. When the knee is extended with the foot externally rotated
and some valgus stress is applied, a click is noted. What is the most likely diagnosis?
Answer: torn medial meniscus
9. Jacob, a 33-year-old construction worker, complains of a "lump on his back" over his scapula.
It has been there for about a year and is getting larger. He says his wife has been able to squeeze
out a cheesy-textured substance on occasion. He worries this may be cancer. When gently
pinched from the side, a prominent dimple forms in the middle of the mass. What is most likely?
Answer: sebaceous cyst
10. A new patient is complaining of severe pruritus that is worse at night. Several family
members also have the same symptom. Upon examination areas of excoriated papules are noted
on some of the interdigital webs of both hands and on the axillae. This finding is consistent with
Answer: Scabies
11. a 58-year-old man comes to your office complaining of bilateral back pain that now awakens
him at night. This has been steadily increasing for the past 2 months. Which one of the following
is the most reassuring in this patient with back pain
Answer: pain that is bilateral term
12. a 32 year-old warehouse worker presents for evaluation of low back pain. He notes a sudden
onset of pain after lifting a set of boxes that were heavier than usual. He also states that he has
numbness and tingling in the left leg. He wants to know if he needs to be off of work. What test
should you perform to assess for herniated disc
Answer: straight- leg raise
13. Mrs. Anderson presents with an itchy rash which is raised and appears and disappears in
various locations. Each lesion lasts for many minutes. What most likely accounts for this rash?
Answer: urticaria or hives
14. a 50-year old woman presents with "left hip pain" of several weeks duration. There is marked
tenderness when you press over her proximal lateral thigh. What do you think she has
Answer: trochanteric bursitis
15. a 28-year old graduate student comes to your clinic for evaluation of pain 'all over." With
further questioning, she is able to relate that the pain is worse in the neck, shoulders, hands and
low back and knees. She denies swelling in her joints. She states that the pain is worse in the
morning. there is no limitation in her range of motion. On physical examination, she has several
points on the muscles of the neck, shoulders, and back that are tender to palpation. Muscle
strength and range of motion are normal. Which one of the following is likely the cause of her
pain
Answer: fibromyalgia
16. a mother brings her 11 month-old to you because her mother-in-law and others have told her
that her baby is jaundiced. She is eating and growing well and performing the developmental
milestones she should for her age. On examination you indeed notice a yellow tone to her skin
from head to toe. Her sclerae are white. To which are should you next questions be related
Answer: diet
17. you are beginning the examination of the skin on a 25-year old teacher. You have previously
elicited that she came to the office for evaluation of fatigue, weight gain, and hair loss. You
strongly suspect that she has hypothyroidism. What is the expected moisture and texture of the
kin of a patient with hypothyroidism.
Answer: dry and rough
18. you are assessing a patient with joint pain and are trying to decide whether it is inflammatory
or non-inflammatory in nature. Which one of the following symptoms is consistent with an
inflammatory process
Answer: tenderness
19. you are assessing a patient with diffuse joint pains and want to make sure that only the joints
are the problem, and that the pain is not related to other diseases. Which of the following is a
systemic cause of joint pain
Answer: lupus
20. 19-year-old college sophomorre comes to the clinic for evaluation of joint pains. The student
has been back from spring break for 2 weeks; during her holiday, she went camping. She notes
that she had a red spot, shaped like a target, but then it started spreading, and then the joint pains
started. she used insect repellent but was in area known to have ticks. She has never been sick
and takes no medications routinely; she has never been sexually active. What is the most likely
cause of her joint pain
Answer: Lyme disease
21. an 85-year-old retired housewife comes with her daughter to establish care. Her daughter is
concerned because her mother has started to fall more. As part of her physical examination, you
ask her to walk across the examination room. Which of the following is not part of the stance
phase of gait?
Answer: foot arched
22. a 33-year-old construction worker comes for evaluation and treatment of acute onset of low
back pain. He notes that the pain is an aching located in the lumbosacral area. It has been present
intermittently for several years; there is no known trauma or injury. He points to the left lower
back. The pain does not radiate and there is numbness or tingling in the legs or incontinence. He
was moving furniture for a friend over the weekend. On physical examination, you note muscle
spasm, with normal deep tendon reflexes and muscle strength. What is the most likely cause of
this patient's low back pain
Answer: mechanical low back pain
23. a 50-year-old realtor comes to your office for evaluation of neck pain. She was in a motor
vehicle collision 2 days ago and assessed by the emergency medical technicians on site, but she
didn't think that she needed to go to the emergency room at that time. Now, she has severe pain
and stiffness in her neck. On physical examination, you note pain and spasm over the
paraspinous muscles on the left side of the neck, and pain when you make the patient do active
range of motion of the cervical spine. What is the most likely cause of this neck pain
Answer: cervical sprain
24. a 68-year-old retired banker comes to your clinic for evaluation of left shoulder pain. He
swims for 30 minutes daily, early in the morning. He notes a sharp, catching pain and a sensation
of something grating when he tries overhead movements of his arm. On physical examination,
you note tenderness just below the tip of the acromion in the area of the tendon insertions. The
drop arm test is negative, and there is no limitation with shoulder shrug. The patient is not
holding his arm close to his side, and there is no tenderness to palpation in the bicipital groove
when the arm is at the patient's side. flexed to 90 degrees, and then supinated against resistance.
Based on this description, what is the most likely cause of his shoulder pain
Answer: rotator cuff tendinitis
25. a high school soccer player "blew out his knee" when the opposing goalie's head and
shoulder struck his flexed knee while the goalie was diving for the ball. All of the following
structures were involved in some way in his injury, but which of the following is actually an
extra-articular structure
Answer: tendons
26. Ray works a physical job and notes pain when he attempts to lift his arm over his head.
When you move the shoulder passively, he has full range of motion without pain and there is no
gross swelling or tenderness. What type of joint disease does this most likely represent?
Answer: extra-articular joint disease
27. Mark is a contractor who recently injured his back. He was told he had a "bulging disc" to
account for the burning pain down his right leg and slight foot drop. The vertebral bodies of the
spine involve which type of joint?
Answer: cartilaginous
28. Which of the following synovial joints would be example of a condylar joint
Answer: temporomandibular joint
29. 58-year-old man comes to your office complaining of bilateral back pain that now awakens
him at night. This has been steadily increasing for the past 2 months. Which one of the following
is the most reassuring in this patient with back pain
Answer: pain that is bilateral
30. Marion presents to your office with back pain associated with constipation and urinary
retention. Which of the following is most likely?
Answer: cauda equina
31. Louise, a 60-year-old, complains of left knee pain associated with tenderness throughout,
redness, and warmth over the joint. Which of the following is least helpful in determining if a
joint problem is inflammatory?
Answer: pain
32. You are working in a college health clinic and seeing a young woman with red, painful,
swollen DIP joint on the left index finger. There are also a few papules, pustules, and vesicles on
reddened bases, located on the distal extremities. This would be consistent with which of the
following
Answer: gonoccocal arthritis
33. An obese 55-year-old woman went through menarche at age 16 and menopause 2 years ago.
She is concerned because an aunt had severe osteoporosis. Which of the following is a risk factor
for osteoporosis?
Answer: delayed menarchy
34. a 38-year-old woman comes to you and has multiple small joint involved with pain, swelling
and stiffness. Which of the following is the most likely explanation
Answer: rheumatoid arthritis